LSAT and Law School Admissions Forum

Get expert LSAT preparation and law school admissions advice from PowerScore Test Preparation.

User avatar
 queenbee
  • Posts: 75
  • Joined: Sep 18, 2022
|
#98042
hi
i changed by answer from B to C because i wasnt sure how you could tell if a mouse was stressed. It seemed to far fetched. I didnt like C either becasue it was a temporary memory impairment....but i selected it only becasue i didnt know if it was possible to determine the stress levels of a mouse.
Any advise on this?
Thanks
 Robert Carroll
PowerScore Staff
  • PowerScore Staff
  • Posts: 1819
  • Joined: Dec 06, 2013
|
#98460
queenbee,

If it's not possible to tell whether a mouse is stressed, that's devastating for the argument! You'd then not know how stressed a mouse was, so the author's entire argument would collapse. So that seems to be more of an argument for answer choice (B) than anything else available.

Robert Carroll
User avatar
 Dancingbambarina
  • Posts: 163
  • Joined: Mar 30, 2024
|
#111453
Jon makes reference to A supporting the conclusion instead of weakening it.

Is it common sense to assume that a higher dose of something still has the same effect? If so, I can see how A is a strengthener. But a much higher doser implies causing other effects (common sense assumption) that are not the same as intended healthily.

so does A help the argument becasue a too-strong dose would mean the same effect as the dose required to lower stress?

The same concept here is present in the LRB example with the molecules changing shape with normal motion (the Rhodopsin example)

There is an assumption that the more of something that causes a reaction, the reaction will increase proportionately. Surely this is non-sensical because the increase of this Ginkgo dose, as with the increase of "normal" motion in the Rhodopsin e.g., is not guaranteed to have the same effect.
 Adam Tyson
PowerScore Staff
  • PowerScore Staff
  • Posts: 5538
  • Joined: Apr 14, 2011
|
#111917
Jon said that "if anything," it strengthens the argument. It depends on how you interpret it. Note that the answer does not say the does is "too high," as you paraphrased it. It just says that it is "significantly higher." So, I see three possible interpretations of answer A;

1. Higher does means greater effect. Could be true, although not guaranteed. This one appeals to my common sense a little bit, even though I wouldn't make a similar inference about, say, pain relievers or antibiotics. This would, if true, strengthen the argument, as Jon suggested.
2. More has no additional effect. You can only get so much stress reduction. Also reasonable, but also not guaranteed. This has no impact on the argument, neither strengthening nor weakening.
3. More is too much and counteracts the benefits of the lower dose. While possible, there's no reason to believe this, and to interpret in this way requires making some outside assumptions that we just shouldn't be making. Yes, if we make these assumptions, then this would weaken the argument, but if we just stick to what it says, then we shouldn't go there.

Good answers don't need this kind of help. You shouldn't have to work hard to make an answer right, and you shouldn't have to guess what the effect in this case would be or choose between equally reasonable alternatives. It should be plain and clear.

And then, of course, there's the correct answer, answer B. If that's true, then the conclusion based on the experiment makes no sense. This answer destroys the argument without needing any help from us in the form of assumptions or outside knowledge. So even if we think one possible interpretation of answer A might weaken the argument, there's no way it can be the answer that does the most to weaken it.

If you find yourself thinking "well, if you interpret it this way, then it could work," then you're almost certainly looking at a wrong answer. The test is simpler than that. Not easy, mind you, but simple.
 Adam Tyson
PowerScore Staff
  • PowerScore Staff
  • Posts: 5538
  • Joined: Apr 14, 2011
|
#111919
I'll add this: the problem with answer A is that we just can't know whether it strengthens, weakens, or does nothing. It can't be a good answer if we can't be sure what it's doing!
User avatar
 Dancingbambarina
  • Posts: 163
  • Joined: Mar 30, 2024
|
#111940
Adam Tyson wrote: Sat Feb 15, 2025 8:05 pm I'll add this: the problem with answer A is that we just can't know whether it strengthens, weakens, or does nothing. It can't be a good answer if we can't be sure what it's doing!
Hi Adam,

Thank you so very much for your response. I'm very grateful for the help and will implement the changes to my assumption-making on this test.
I understand perfectly and thanks again for all the help, on this question and the others. I am working hard and will make you all proud.

Get the most out of your LSAT Prep Plus subscription.

Analyze and track your performance with our Testing and Analytics Package.